The Student Room Group

Confused about limits in an improper integral

Hi, would somebody kindly explain how in this question

the limits on the integral with the substitution have become 1 and 0? And how do I answer the last question, given that the answer to part (b)(ii) is -1/2?

Thanks
Reply 1
Original post by foorganders
Hi, would somebody kindly explain how in this question

the limits on the integral with the substitution have become 1 and 0? And how do I answer the last question, given that the answer to part (b)(ii) is -1/2?

Thanks


the reciprocal of 1 is 1
the reciprocal of infinity is zero (loosely before I get lectured...)
Reply 2
Original post by TeeEm
the reciprocal of 1 is 1
the reciprocal of infinity is zero (loosely before I get lectured...)


Ok, why have the upper and lower limits swapped?
Reply 3
Original post by foorganders
Ok, why have the upper and lower limits swapped?


they used the minus from the substitution to reverse the limits
Reply 4
Original post by TeeEm
they used the minus from the substitution to reverse the limits

I'm not sure what you mean, when I substituted x I didn't have a minus, or rather I had two.
(edited 9 years ago)
Reply 5
Original post by foorganders
I'm not sure what you mean, when I substituted x I didn't have a minus, or rather I had two.


the integral of f(x) with limits x=a to x=b

is the same as

minus the integral of f(x) with limits x=b to x=a
Reply 6
Original post by TeeEm
the integral of f(x) with limits x=a to x=b

is the same as

minus the integral of f(x) with limits x=b to x=a


Ok. Is that relevant to the fact that it's a reciprocal? I still can't see why they have done that
Reply 7
Original post by foorganders
Ok. Is that relevant to the fact that it's a reciprocal? I still can't see why they have done that


nothing to do with reciprocals/it is always true

when find dy/dx to do the substitution you generate a minus which you can "throw" it away by reversing the limits
Reply 8
Original post by TeeEm
nothing to do with reciprocals/it is always true

when find dy/dx to do the substitution you generate a minus which you can "throw" it away by reversing the limits

Right ok I get that but in the substitution you don't get a minus because when you substitute 1/y into the ln part you get ln(y^-2) or -ln(y^2) so the two minuses cancel
Reply 9
Original post by foorganders
Right ok I get that but in the substitution you don't get a minus because when you substitute 1/y into the ln part you get ln(y^-2) or -ln(y^2) so the two minuses cancel


y = 1/x

dy/dx = - 1/x2

there is a minus and nothing cancels it
Original post by TeeEm
y = 1/x

dy/dx = - 1/x2

there is a minus and nothing cancels it


I know that there's a minus there, what I'm saying is that lnx2=ln1y2=lny2\ln x^2 = \ln \frac{1}{y^2} = - \ln y^2
Reply 11
Original post by foorganders
I know that there's a minus there, what I'm saying is that lnx2=ln1y2=lny2\ln x^2 = \ln \frac{1}{y^2} = - \ln y^2


nowhere does it say in the question that the (i) is equal to (ii)
Reply 12
Original post by foorganders
I know that there's a minus there, what I'm saying is that lnx2=ln1y2=lny2\ln x^2 = \ln \frac{1}{y^2} = - \ln y^2



012ylnydy=1lnx2x3dx\displaystyle \int_0^1 2y \ln y \, \mathrm{d}y = -\int_{1}^{\infty}\frac{\ln x^2}{x^3} \, \mathrm{d}x
Original post by Zacken
012ylnydy=1lnx2x3dx\displaystyle \int_0^1 2y \ln y \, \mathrm{d}y = -\int_{1}^{\infty}\frac{\ln x^2}{x^3} \, \mathrm{d}x


Sorry but I'm not convinced that this is correct. Unless I've got something wrong, but my working is:

x=1y[br][br]dx=1y2dy[br]lnx2x3dx[br]=lny2y1dy[br]=2ylnydy[br]x=\dfrac{1}{y} [br][br]dx=-\dfrac {1}{y^2} dy[br]\int\dfrac{\ln x^2}{x^3}dx[br]=\int -\dfrac{\ln y^{-2}}{y^{-1}}dy[br]=\int 2y\ln ydy[br]
Reply 14
Original post by foorganders
Sorry but I'm not convinced that this is correct. Unless I've got something wrong, but my working is:

x=1y[br][br]dx=1y2dy[br]lnx2x3dx[br]=lny2y1dy[br]=2ylnydy[br]x=\dfrac{1}{y} [br][br]dx=-\dfrac {1}{y^2} dy[br]\int\dfrac{\ln x^2}{x^3}dx[br]=\int -\dfrac{\ln y^{-2}}{y^{-1}}dy[br]=\int 2y\ln ydy[br]


Yes, I agree. I'm saying the integral from 0 to 1 of blah is NOT equal to the original integral. It never said so in the question. It is in fact equal to negative the original integral.
Original post by Zacken
Yes, I agree. I'm saying the integral from 0 to 1 of blah is NOT equal to the original integral. It never said so in the question. It is in fact equal to negative the original integral.


OH. OH! I understand now! so in the final question the value of that is equal to the negative of part (ii), yes?
Reply 16
Original post by foorganders
OH. OH! I understand now! so in the final question the value of that is equal to the negative of part (ii), yes?


Yes! :smile:
Original post by Zacken
Yes! :smile:


Ah! Thank you!

Quick Reply

Latest